Đến nội dung

congchuasaobang nội dung

Có 53 mục bởi congchuasaobang (Tìm giới hạn từ 07-06-2020)



Sắp theo                Sắp xếp  

#483370 tìm Min của $y= \sqrt{x^{2}+x+1}+\sqrt{x^{2}-x+1}$

Đã gửi bởi congchuasaobang on 16-02-2014 - 00:53 trong Bất đẳng thức và cực trị

câu 1 : a,Tìm giá trị nhỏ nhất của hàm số : y = $\sqrt{x^{2}+x+1}+\sqrt{x^{2}-x+1}$

            b, Cho 3 số thực x,y,z đều lớn hơn 2 và thỏa mãn điều kiện : $\frac{1}{x}+\frac{1}{y}+\frac{1}{z}= 1$

                          Chứng minh rằng : (x-2)(y-2)(z-2)$\leq$ 1

 

câu 2 : cho tam giác ABC nhọn và O là một điểm nằm trong tam giác. Các tia AO, BO, CO lần lượt cắt BC, AC, AB tại M, N, P

                     Chứng minh : $\frac{AM}{OM}+\frac{BN}{ON}+\frac{CP}{OP}\geq 9$

 

câu 3 : cho a, b, c là 3 số thực dương.

                      Chứng minh rằng: $\sqrt{\frac{a}{b+c+2a}}+\sqrt{\frac{b}{c+a+2b}}+\sqrt{\frac{c}{a+b+2c}}\leq \frac{3}{2}$

 

câu 4 : cho tam giác ABC có độ dài 3 cạnh AB=c; BC=a; CA=b, Các góc của tam giác đó thỏa mãn: $\widehat{C}=2\widehat{A}+\widehat{B}$. Chứng minh rằng : $c^{2}< 2a^{2}+b^{2}$

 

@Viet Hoang 99: Chú ý tiêu đề




#488097 Tính $\sqrt[3]{20+14\sqrt{2}}+\sqrt[3...

Đã gửi bởi congchuasaobang on 21-03-2014 - 19:15 trong Đại số

Tính

$\sqrt[3]{20+14\sqrt{2}}+\sqrt[3]{20-14\sqrt{2}}$

Để mình làm 1 bài đầy đủ luôn cho :icon6:  :icon6:  :icon6:

 Ta có A = $\sqrt[3]{20+14\sqrt{2}}+\sqrt[3]{20-14\sqrt{2}}$

   nên $A^{3}=(\sqrt[3]{20+14\sqrt{2}}+\sqrt[3]{20-14\sqrt{2}})^{3}$

                    = $20+14\sqrt{2}+20-14\sqrt{2}+3\sqrt[3]{(20+14\sqrt{2})(20-14\sqrt{2})}(\sqrt[3]{20+14\sqrt{2}}+\sqrt[3]{20-14\sqrt{2}})$

                    = 40+3A.2

                    = 40+6A

do đó $A^{3}-6A-40=0$

nên $(A-4)(A^{2}+4A+10)=0$              (1)

  ta có $A^{2}+4A+10= (A+2)^{2}+6\geq 6$ với mọi A

 do đó từ (1) ta được A-4=0 nên A=4

      Vậy  $\sqrt[3]{20+14\sqrt{2}}+\sqrt[3]{20-14\sqrt{2}}$ =4  :)  :)  :)




#483472 Giải phương trình : $\sqrt[3]{x+1}=x^{3}-15x^...

Đã gửi bởi congchuasaobang on 16-02-2014 - 16:39 trong Phương trình, hệ phương trình và bất phương trình

câu 1 : Giải phương trình : $\sqrt[3]{x+1}=x^{3}-15x^{2}+75x-131$

 

câu 2 : Cho phương trình : $2x^{2}-x-2=0$ có các nghiệm là x1, x2. Không giải phương trình hãy thực hiện: 

             a, Tính giá trị của biểu thức: A= $\frac{x1^{2}}{x2+1}+\frac{x2^{2}}{x1+1}$

             b, Lập một phương trình bậc 2 theo y có 2 nghiệm là

                             $y1=x1+\frac{2}{x2}; y2=x2+\frac{2}{x1}$

 

câu 3 : a, Tìm các cặp số nguyên x, y thỏa mãn:

                         $(x-2006)^{2}= y(y+1)(y+2)(y+3)$

           b, Giải hệ phương trình x+y+z=6

                                          và   xy+yz-zx=7

                                         và $x^{2}+y^{2}+z^{2}=14$

 

câu 4 : a, Giải phương trình : $\sqrt{x-2}+\sqrt{6-x}= \sqrt{x^{2}-8x+24}$

            b, Giải hệ phương trình : $x+y +\frac{1}{x}+\frac{1}{y}=\frac{9}{2}$                   

                                             và   $xy + \frac{1}{xy}=\frac{5}{2}$

  




#483540 Giải phương trình : $\sqrt[3]{x+1}=x^{3}-15x^...

Đã gửi bởi congchuasaobang on 16-02-2014 - 20:50 trong Phương trình, hệ phương trình và bất phương trình

a, Theo BĐT Bunhiacopxki: $(\sqrt{x-2}+\sqrt{6-x})^2\leq 2.4\Leftrightarrow\sqrt{x-2}+\sqrt{6-x}\leq \sqrt{8}$

$\sqrt{x^2-8x+24}=\sqrt{(x-4)^2+8}\geq \sqrt{8}$. Dâu bằng xảy ra khi x=4

bạn làm rõ hơn ở chỗ BĐT Bunhiacopxki dc ko ????




#507874 Cho các số dương a,b,c biết: $\frac{a}{1+a}+\frac{b}{1+b}+...

Đã gửi bởi congchuasaobang on 19-06-2014 - 19:30 trong Bất đẳng thức và cực trị

Cho các số dương a,b,c biết: $\frac{a}{1+a}+\frac{b}{1+b}+\frac{c}{1+c}\leq 1$. Chứng minh rằng $abc\leq \frac{1}{8}$




#507888 Cho các số dương a,b,c biết: $\frac{a}{1+a}+\frac{b}{1+b}+...

Đã gửi bởi congchuasaobang on 19-06-2014 - 20:48 trong Bất đẳng thức và cực trị

mọi người giúp bài này nữa luôn ạ 

   Cho 3 số thực dương a,b,c Chứng minh:

a, $\frac{a}{b+c}+\frac{b}{c+a}+\frac{c}{a+b}\geq \frac{3}{2}$

b, $\frac{a}{bc(c+a)}+\frac{b}{ca(a+b)}+\frac{c}{ab(b+c)}\geq \frac{27}{2(a+b+c)}$




#489172 Tìm 2 số nguyên dương $p,q$ sao cho: $p^{2}-q^{...

Đã gửi bởi congchuasaobang on 28-03-2014 - 11:37 trong Tổ hợp - Xác suất và thống kê - Số phức

4 TH luôn hả bạn? :)

2 trường hợp 7;1 vs 1; 7 thôi mà

 Ta có p; q nguyên dương nên p+q >0

xét 2 trường hợp

 +)     p-q=7

         p+q=1

     ta được p=4; q=-3 (loại)

+)      p-q=1

         p+q=7

     ta được p=4 ; q=3

vậy (p;q)=(4;3)




#493628 Chứng minh rằng $PQ^2=QR.ST$

Đã gửi bởi congchuasaobang on 17-04-2014 - 22:47 trong Hình học

a, Ta có $\Delta OAB$ cân ( vì OA, OB là bán kính đường tròn tâm O )

           nên $\widehat{OAB}=\widehat{OBA}$                                                                    (1)

    Xét $\Delta AQR$ vuông tại R, ta có $\widehat{AQR}+\widehat{QAR}=90^{\circ}$

                                                      hay  $\widehat{AQR}+\widehat{OAB}=90^{\circ}$        (2)

    Xét $\Delta BST$ vuông tại T, ta có $\widehat{TSB}+\widehat{SBT}=90^{\circ}$

                                                     hay $\widehat{TSB}+\widehat{OBA}=90^{\circ}$           (3)

    Từ (1), (2) và (3) ta được $\widehat{TSB}=\widehat{AQR}$

                                 nên $\widehat{PSQ}=\widehat{PQS}$ ( 2 cặp góc đối đỉnh bằng nhau)

               hay $\Delta PQS$ cân tại P




#487762 $x^4-mx^3-(2m+1)x^2+mx+1=0$

Đã gửi bởi congchuasaobang on 19-03-2014 - 12:53 trong Phương trình, hệ phương trình và bất phương trình

ta có pt có 2 nghiệm phân biệt lớn hơn 1 khi 1<x1<x2

                     $\Delta >0$

                     $x1-1+x2-1> 0$

                      $(x1-1)(x2-1)> 0$




#532389 hợp số

Đã gửi bởi congchuasaobang on 08-11-2014 - 20:01 trong Số học

câu 1: chứng minh 2^10+5^12 là hợp số

 

câu 2 tìm x, y, z biết xyy1+4z=z^2




#532452 hợp số

Đã gửi bởi congchuasaobang on 08-11-2014 - 23:30 trong Số học

Câu 1: 

$ 2 ^{10} + 5^{12} = 15657^2 - 1000^2$

Vì vậy nó là hợp số

bạn làm kĩ hơn được không??? mình không hiều




#532458 hợp số

Đã gửi bởi congchuasaobang on 08-11-2014 - 23:59 trong Số học

Bạn tách hằng đẳng thức ra. Đó là hợp số vì lúc đó nó có hai ước số khác 1 và khác chính nó nữa.

 

$ 15657 + 1000 = 16657 $

$ 15657 - 1000 = 14657 $

không phải, ý mình là từ đề bài làm sao để biến đổi thành $15657^{2}-1000^{2}$ í ??????




#535578 Chứng minh rằng $\widehat{BDM}= \widehat{CDN...

Đã gửi bởi congchuasaobang on 30-11-2014 - 17:56 trong Hình học

1/ Cho $\Delta ABC$ nhọn, có 2 đường cao BE và CF. Gọi P, Q lần lượt là hình chiếu vuông góc của B và C trên EF. C/m Sbpqc = S$\Delta BCE$ + S$\Delta BFC$

2/ Cho $\Delta ABC$ có CB là phân giác của $\widehat{ACD}$. Lấy M, N thuộc đoạn CB sao cho $\widehat{BAM}=\widehat{CAN}$. Chứng minh rằng $\widehat{BDM}= \widehat{CDN}$

3/ Cho $\Delta ABC$, trung tuyến AM. Gọi D là điểm trên đoạn BM. Kẻ đường thẳng qua M song song AD cắt AC tại E. C/m $\frac{S\Delta DEC}{S\Delta ABC}=\frac{1}{2}$




#500415 hệ phương trình :$\left\{\begin{matrix} 3x...

Đã gửi bởi congchuasaobang on 20-05-2014 - 23:38 trong Phương trình, hệ phương trình và bất phương trình

Xét hệ phương trình :$\left\{\begin{matrix} 3x-my=x^{2} & \\ 3y-mx=y^2& \end{matrix}\right.$

  a, Giải hệ phương trình khi m=1

  b, Chứng minh rằng nếu m>1 thì hệ đang xét không thể có nghiệm thỏa mãn điều kiện $x\neq y$

 




#500414 Chứng tỏ rằng: $a^{3}-b^{3}+c^{3}+3abc= (a...

Đã gửi bởi congchuasaobang on 20-05-2014 - 23:32 trong Đại số

a, Chứng tỏ rằng: $a^{3}-b^{3}+c^{3}+3abc= (a-b+c)(a^{2}+b^{2}+c^{2}+ab+bc-ca)$ với mọi số thực a,b,c

b, Chứng minh nếu d,e,f là các số nguyên thỏa mãn $d=e\sqrt[3]{2}+f\sqrt[3]{4}=0$ thì d=e=f

c, Tìm các số hữu tỉ p,q,r để có đẳng thức $\frac{3-3\sqrt[3]{4}}{1-\sqrt[3]{2}+\sqrt[3]{4}}= p+q\sqrt[3]{2}+r\sqrt[3]{4}$




#489145 Vấn đề về phương trình và phương trình nghiệm nguyên [sharedmedia=core:attach...

Đã gửi bởi congchuasaobang on 27-03-2014 - 22:40 trong Đại số

 

1.Giải phương trình

2.Tìm nghiệm nguyên của phương trình 

 

 câu 2 : http://diendantoanho...2-6xy-13y2-100/




#537215 Chứng minh rằng $(a^{2}+b^{2})(x^{2}+y^...

Đã gửi bởi congchuasaobang on 11-12-2014 - 18:10 trong Đại số

1/ Biết x, y, z liên hệ với nhau bởi các đẳng thức: $x^{2}-y=a; y^{2}-z=b; z^{2}-x=c$

Tính giá trị của biểu thức: P= $x^{3}(z-y^{2})+ y^{3}(x-z^{2})+z^{3}(y-x^{2})á+xyz(xyz-1)$

2/ Biết x+y=a+b; $x^{2}+y^{2}=a^{2}+b^{2}$

C/m: $x^{n}+y^{n}=a^{n}+b^{n}$, với n nguyên dương

3/Tìm các nghiệm nguyên của phương trình: $4x^{4}+8x^{2}y+3y^{2}-4y-15=0$

4/C/m: 

a/ $(a^{2}+b^{2})(x^{2}+y^{2})=(ax-by)^{2}+(bx+ay)^{2}$

b/ $(a^{2}+b^{2}+c^{2})(x^{2}+y^{2}+z^{2})-(ax+by+cz)^{2}=(bx-ay)^{2}+(cy-bz)^{2}+(ax-cz)^{2}$

 




#486863 tìm k để S tam giác MNP = 5/8 S tam giác ABC

Đã gửi bởi congchuasaobang on 14-03-2014 - 21:38 trong Hình học

bx mi giỏi rồi đó, tau mà bx đứa mô thì ĐỪNG COK TRÁCH :angry:  :angry:  <_<  <_<  :angry:  :angry: giỏi thì làm đi, khoeeeeeeeeeeeeeeeeeeeee, hứ




#481637 tìm k để S tam giác MNP = 5/8 S tam giác ABC

Đã gửi bởi congchuasaobang on 07-02-2014 - 16:28 trong Hình học

Câu 1: Cho nửa đường tròn đường kính AB=2R. Gọi C là điểm tùy ý trên nửa đường tròn, D là hình chiếu vuông góc của C lên AB. Tia phân giác góc ACD cắt đường tròn đường kính AC tại điểm thứ hai là E, cắt tia phân giác góc ABC tại H

                  a, C/m: AE // BH

                  b, Tia phân giác góc CAB cắt đường tròn đường kính AC tại điểm thứ hai là F, cắt  CE tại I. Tính diện tích $\Delta FID$ trong trường hợp tam giác đó đều? ( Với $sin 15^{\circ} \approx 0,2588$ )

                  c, Trên đoạn BH lấy điểm K sao cho HK = HD, gọi J là giao điểm của AF và BH. Xác định ví trí của C để tổng các khoảng cách từ các điểm I, J, K đến đường thẳng AB đạt giá trị lớn nhất

 

 

Câu 2: Trên 3 cạnh  AB, BC, CA của tam giác ABC lần lượt lấy 3 điểm M, N, P sao cho $\frac{AM}{MB}=\frac{BN}{NC}=\frac{CP}{PA}=k$. Tìm k để $S\Delta MNP=\frac{5}{8}S\Delta ABC$

 

 

:icon13:  :icon13:  :icon13:




#541774 Cho x,y,z đôi một khác nhau và $\frac{1}{x}+...

Đã gửi bởi congchuasaobang on 25-01-2015 - 01:26 trong Đại số

Câu 1: Cho A= $\left ( \frac{2x-x^2}{2x^2+8} -\frac{2x^2}{x^3-2x^2+4x+8}\right ).\left ( \frac{2}{x^2}+\frac{1-x}{x} \right )$

   a, Rút gọn A

   b, Tìm x để A nguyên

 

Câu 2: Cho x,y,z đôi một khác nhau và $\frac{1}{x}+\frac{1}{y}+\frac{1}{z}=0$

       Tính Q= $\frac{yz}{x^2+2yz}+\frac{xz}{y^2+2xz}+\frac{xy}{z^2+2xy}$




#477883 bất đẳng thức lớp 9

Đã gửi bởi congchuasaobang on 18-01-2014 - 18:32 trong Bất đẳng thức và cực trị

Câu 1 : Cho 3 số thực dương a,b,c thỏa mãn a+b+c=1 

                    Chứng minh : $\frac{1}{ac}$ + $\frac{1}{bc}\geq$ 16

Câu 2 : cho a,b,c là các số thực dương 

                    Chứng minh rằng $\frac{a}{b}+\frac{b}{c}+\frac{c}{a}\geq \frac{a+2012}{b+2012}+\frac{b+2012}{c+2012}+\frac{c+2012}{a+2012}$




#478383 chứng minh các đẳng thức trong tứ giác nội tiếp đường tròn

Đã gửi bởi congchuasaobang on 21-01-2014 - 20:50 trong Hình học

cho tứ giác ABCD nội tiếp đường tròn (O) , Các cạnh đối AB và CD kéo dài cắt nhau tại E, các cạnh đối AD và CB cắt nhau tại F. Phân giác trong của góc DFC cắt AB tại P, cắt CD tại Q

     a, chứng minh PE=QE

     b, chứng minh $EF^{2}= FA.FD + EA.EB$ :excl:  :excl:  :excl:




#479477 giải phương trình với 2 ẩn x,y

Đã gửi bởi congchuasaobang on 27-01-2014 - 21:09 trong Đại số

giải hệ phương trình

     $x^{3}+3xy^{2}=-49$

và$x^{2}-8xy+y^{2}=8y-17x$

:icon6:  :icon6:  :icon6:  :icon6:




#500566 Với a,b,c là các số thực dương. Chứng minh rằng $A+B\geq C+D$...

Đã gửi bởi congchuasaobang on 21-05-2014 - 19:01 trong Bất đẳng thức và cực trị

Với a,b,c là các số thực dương. Chứng minh rằng $A+B\geq C+D$ với:

    A=$\frac{1}{a(1+b)}+\frac{1}{b(1+c)}+\frac{1}{c(1+a)}$

    B=$\frac{ab}{1+a}+\frac{bc}{1+b}+\frac{ca}{1+c}$

    C=$\frac{1}{1+a}+\frac{1}{1+b}+\frac{1}{1+c}$

    D= $\frac{b}{1+b}+\frac{c}{1+c}+\frac{a}{1+a}$




#481088 chứng minh một tam giác có diện tích bằng 1 không thể chứa trong một hình bìn...

Đã gửi bởi congchuasaobang on 05-02-2014 - 11:50 trong Hình học

Câu 1: Cho một góc nhọn và A là một điểm nằm trong góc đó. Hãy dựng một tam giác ABC có chu vi bé nhất mà các đỉnh B, C tương ứng nằm trên 2 cạnh của góc đó

Câu 2: Cho tam giác ABC có 3 góc nhọn nội tiếp trong đường tròn ( O;R). Điểm M di động trên cung nhỏ Bc. Từ M kẻ các đường thẳng MH, MK lần lượt vuông góc với AB, AC ( H thuộc đường thẳng AB, K thuộc đường thẳng AC)

                  a, C/m 2 tam giác MBC và MHK đồng dạng

                  b, Tìm vị trí của M để độ dài đoạn HK lớn nhất

Câu 3: Chứng minh rằng khoảng cách từ điểm P được chọn bất kì trong hình bình hành đó không vượt quá bán kính R của đường tròn ngoại tiếp tam giác ABC

Câu 4: chứng minh một tam giác có diện tích bằng 1 không thể chứa trong một hình bình hành có diện tích nhỏ hơn 2

:icon13:  :wacko:  :icon14: